LSAT and Law School Admissions Forum

Get expert LSAT preparation and law school admissions advice from PowerScore Test Preparation.

User avatar
 Dave Killoran
PowerScore Staff
  • PowerScore Staff
  • Posts: 5852
  • Joined: Mar 25, 2011
|
#27112
Complete Question Explanation
(The complete setup for this game can be found here: lsat/viewtopic.php?t=49)

The correct answer choice is (E)

The answer requires you to find a grouping that violates at least one of the rules. As you might expect, the more fruits listed in an answer choice, the more likely that the answer will violate one or more of the rules. Answer choice (E) lists the greatest number of fruits, and violates two of the rules. First, from the first rule, K and P cannot both be carried by the stand. Second, from the inferences, when the stand carries P it must also carry T. Thus, answer choice (E) cannot be a viable list of fruits carried by the stand, and answer choice (E) is thus correct.
 akandell910
  • Posts: 1
  • Joined: Sep 17, 2019
|
#68229
Hey! I'm confused why A isn't the answer here, since by the second rule, the stand carries kiwis or tangerines but not both. I see why E is also wrong, but I am confused by A is not also wrong here.
 Claire Horan
PowerScore Staff
  • PowerScore Staff
  • Posts: 408
  • Joined: Apr 18, 2016
|
#68249
Hi akandell910,

Thanks for asking your question so clearly--I saw the source of your difficulty right away. The second rule cannot be simplified to mean
the stand carries kiwis or tangerines but not both.
Here's why:
The second rule states,
If the stand does not carry tangerines, then it carries kiwis.


This can be written as T :arrow: K.
That means if T is not present, we know that K is present.
(If T is present, we know nothing, since the sufficient clause is not satisfied. So K could be present or not.)

The contrapositive is K :arrow: T.
That means if K is not present, we know that T is present.
(Again, if K is present, we know nothing, so T could be present or not.)

If we put the original statement and its contrapositive together, we get that T or K (or both) must be present.

Consider a conditional statement from everyday life: "If it rains today, my game will be cancelled." If it does not rain, we know nothing about whether the game is cancelled. The game may happen as scheduled or it may be cancelled for some other reason. When the sufficient clause is not satisfied, we don't have any information. You may want to review conditionals in Lesson 2 if you're not feeling clear enough on this point.

Thanks for the question, and good luck!

Get the most out of your LSAT Prep Plus subscription.

Analyze and track your performance with our Testing and Analytics Package.